¿Las cargas en movimiento y los campos eléctricos cambiantes son causas diferentes?

Pregunta

¿ Son las cargas en movimiento y los campos eléctricos variables en el tiempo causas realmente distintas de los campos magnéticos?


Varios fenómenos EM

Dos propósitos más grandes son proporcionar algunos antecedentes para asegurarme de que sigo las ecuaciones de Maxwell y obtener alguna intuición sobre estas cuatro o cinco leyes aparentemente inconexas, marque cualquier cosa que esté mal:

1. Las cargas eléctricas existen y se atraen/repelen. Podemos llamar a la posible fuerza por unidad de carga de ellos el "campo eléctrico".

No existen "cargas magnéticas" análogas. (Usualmente dicho como "sin monopolos magnéticos")

2. Además de las cargas, también se pueden generar campos eléctricos a partir del campo magnético que cambia en ese punto.

3. Los campos magnéticos ejercen fuerza sobre las cargas en movimiento.

P1: ¿Correcto hasta ahora?


Los importantes

Volviendo a la pregunta principal:

4. (O 4 y 5.) Los campos magnéticos se generan únicamente cambiando el campo eléctrico. Un ejemplo de esto es una carga en movimiento (que obviamente provoca un campo eléctrico cambiante). Una carga puntual en movimiento crea un campo magnético que se puede determinar de la misma manera observando cómo cambia el campo eléctrico de esa carga. Pero con la corriente, el campo eléctrico aparentemente estable e inmutable alrededor de una corriente es en realidad campos nuevos que reemplazan a los que se alejan. Entonces, en realidad son campos en movimiento superpuestos y, por lo tanto, forman un campo magnético.

La pregunta es básicamente, ¿ese párrafo ⬆️ es correcto? (Q2)


¿Cómo se relacionan?

Siéntase libre de agregar cómo pueden ser fenómenos independientes menos totales que los 5 enumerados (4 si tengo razón). Tal vez 1 y 2 provienen de una carga eléctrica estacionaria que es como un campo magnético en movimiento.

P3: Además de si 4 y 5 son realmente iguales o no, ¿hay alguna forma de ver dos o más de estos mecanismos relacionados de alguna manera?

Me acabo de dar cuenta de que mi respuesta tenía un error. Lo corregiré mañana (tengo demasiado sueño ahora), y también verificaré si todo lo que dijiste es correcto.
@Wolphramjonny Ok, muchas gracias. Dormir bien.

Respuestas (2)

Un hecho no tan conocido es que es posible obtener una solución completa para las ecuaciones de Maxwell siempre que asuma que las distribuciones de carga y corriente caen lo suficientemente rápido a medida que avanza hacia el infinito espacial. Estas soluciones son generalizaciones de las leyes de Coulomb y Biot-Savart para casos dependientes del tiempo y se conocen como ecuaciones de Jefimenko. Se presentan en los libros de texto habituales de electromagnetismo y cito aquí sus expresiones del libro de Griffiths (etiquetas según la 4ª edición):

(10.36) mi ( r , t ) = 1 4 π ϵ 0 [ ρ ( r , t r ) R 2 R ^ + ρ ˙ ( r , t r ) C R R ^ j ˙ ( r , t r ) C 2 R ] d τ , (10.38) B ( r , t ) = m 0 4 π [ j ( r , t r ) R 2 + j ˙ ( r , t r ) C R ] × R ^ d τ ,
donde uso unidades SI y denoto R = r r (Griffiths usa una cursiva r ), t r = t R C es el tiempo retardado.

Estas ecuaciones no son particularmente convenientes para el cálculo directo (las integrales pueden volverse bastante engorrosas), pero hacen que algunas consideraciones fenomenológicas sean mucho más claras. Por ejemplo, observe la dependencia de los campos en la derivada temporal de la corriente: confirma que un campo eléctrico cambiante no es realmente la causa de la inducción de un campo magnético. De hecho, el cambio en la corriente que necesita para cambiar el campo eléctrico es el mismo que necesitaría para inducir el campo magnético correspondiente. Es una especie de coincidencia, por así decirlo, no una relación de causa-consecuencia. Las ecuaciones de Maxwell realmente no pueden hacer esta distinción, pero una vez que se resuelven, puedes verlo directamente a partir de las expresiones.

Ante esto, permítanme tratar de abordar cada una de sus preguntas.

Q1

  1. A nivel de Electrodinámica Clásica, esto es correcto, pero debo mencionar que los monopolos magnéticos son una línea activa de investigación tanto en Física de Partículas como en Física de Materia Condensada.
  2. Algo así como. Observamos esto, pero es por pura coincidencia. Fundamentalmente, los campos electromagnéticos son generados por cargas y corrientes, a veces por medio de sus derivadas temporales.
  3. Correcto.

Q2

  1. En ausencia de monopolos magnéticos, los campos magnéticos son causados ​​por corrientes eléctricas, como la de una carga en movimiento. El campo eléctrico causado por una corriente se debe a la densidad de carga, su derivada en el tiempo y la derivada en el tiempo de la corriente. La misma corriente que genera un campo eléctrico genera un campo magnético y, a menudo, pueden confundirse como la causa del otro.

Q3

Prefiero ver las ecuaciones de Maxwell de otra forma. Como mencionaste, existen cargas eléctricas y están sujetas a fuerzas relacionadas con campos eléctricos y magnéticos. Esta es mi visión de la ley de fuerza de Lorentz. Entonces las ecuaciones de Maxwell dicen

  1. Dondequiera que haya carga, el campo eléctrico diverge o converge, haciendo que las cargas cercanas se atraigan o repelan según sus signos.
  2. Las líneas de campo magnético siempre están cerradas. No hay fuentes ni drenajes de campo magnético.
  3. El campo eléctrico siempre se enrolla alrededor de una variación del campo magnético. De las ecuaciones de Jefimenko sabemos que esto no es una relación de causa-consecuencia, sino más bien una especie de coincidencia.
  4. El campo magnético se enrosca alrededor de las corrientes y los cambios en el campo eléctrico. Mientras que el primero es un efecto de causa-consecuencia (la corriente genera el campo que se enrosca a su alrededor), el segundo no lo es: es una coincidencia debido a los cambios en la corriente que genera tanto el cambio en el campo eléctrico como en el magnético. (lo sabemos por las ecuaciones de Jefimenko).
@AlBrown He actualizado mi respuesta en consecuencia. Por favor, échale otro vistazo
Wow, escribiste que los campos magnéticos y eléctricos son causados ​​por cargas y corrientes y sus derivados. ¿Podríamos hacer las ecuaciones sin campos como variables dependientes? Oh, eso es lo que es jeffemenk. Muchas gracias
@AlBrown Sí. Al hacer cálculos, no hay problema en asumir este punto de vista. Produce los resultados correctos y hace que las cosas sean bastante más simples de calcular, diría yo.
¿Pregunta después de todo? 1. Esperando usar jefimenko. Estamos obteniendo E y B en el punto r , tiempo t, integrando sobre cualquier otro punto r r en R 3 (como si r was (0,0,0) es una forma de decirlo). Entonces, ¿la integral es solo una integral de volumen? En wikipeda tienen [ . . . ]   d 3 r y escribiste de griffiths d τ ' . nunca he visto d 3 o d τ . ¿Los tres son iguales (incluyendo "una integral de volumen")? Así que en realidad podríamos tener   [   ( . . . d X )   d y ]   d z dependiendo de la forma funcional? Gracias 2. El retraso de tiempo significa que sería lo que la configuración no era. ¿Por qué funciona eso?
@AlBrown Ambos d τ y d 3 r Integral de volumen medio. El tiempo de retraso significa que debe integrar la configuración anterior, no la actual. Eso se debe a la velocidad finita de la luz, que ya es una característica de la electrodinámica clásica. La suposición de caer lo suficientemente rápido está ahí para que las integrales converjan. si eliges ρ ( r , t ) = constante , las integrales serán divergentes. Afortunadamente, las situaciones de interés físico a menudo (si no siempre) tienen cargas que se extienden solo por una porción finita del espacio.
Gracias de nuevo por toda la información. Solo si está interesado, agregué una respuesta. Es posible que le guste solo la primera parte que intenta resumir a Maxwell en la línea que dijo

Extrayendo algunos aspectos destacados de la respuesta detallada de Alves:

P3: Ecuaciones de Maxwell

Las cuatro ecuaciones de Maxwell (cinco relaciones) se entienden mejor como:


1. Las cargas eléctricas provocan campos eléctricos que convergen/divergen hacia/desde la carga:

mi = ρ ε 0

(En otras palabras, las cargas se atraen/repelen: F 1 , 2 = k mi q 1 q 2 r 2 )


2. Las corrientes provocan campos magnéticos que se enroscan alrededor de la corriente:

× B = m 0 j       ( + m 0 ε 0 mi t )

(En otras palabras, las corrientes se atraen/repelen: F 1 , 2 = m 0 I 1 I 2 L 2 π r )


3. Las líneas de campo magnético siempre están cerradas, sin fuentes ni drenajes de campo:

B = 0

(Bastante sencillo hasta aquí.)


4. El campo eléctrico se enrosca alrededor de los cambios en el campo magnético:

× mi = B t

Esta es una consecuencia, pero durante Maxwell se considera una relación adicional.


5. El campo magnético se enrosca alrededor de los cambios en el campo eléctrico:

× B = m 0 ε 0 mi t       ( + m 0 j )

Esta es una consecuencia, pero durante Maxwell se considera una relación adicional.


De las ecuaciones de Jefimenko sabemos que 4., 5. no son causales, ni del rotacional a la derivada ni viceversa. Los plazos se deben a las variaciones actuales que afectan a cada campo individualmente. Pero sin campos, el análisis es más difícil. Si se utilizan campos (Maxwell), se deben incluir los términos en 2 y 5.



Q1

A. Las cargas eléctricas existen y se atraen/repelen. Podemos llamar a la posible fuerza por unidad de carga de ellos, el "campo eléctrico".

Cierto , y además las corrientes también se atraen/repelen. Podemos llamar a la posible fuerza de una corriente sobre otra corriente si estuviera presente, el "campo magnético".

B. ¿No existen “cargas magnéticas” monopolares?

Parcialmente cierto . Normalmente sí, y hasta hace poco era cierto en todas partes. Ahora los monopolos magnéticos son una línea activa de investigación tanto en Física de Partículas como en Física de Materia Condensada.

C. ¿Además de las cargas, los campos eléctricos también pueden generarse a partir del cambio del campo magnético en ese punto?

Técnicamente no es cierto Ver Q3

D. Los campos magnéticos ejercen fuerza sobre las cargas en movimiento.

Cierto, y como causa directa. Las corrientes ponen fuerzas sobre otras corrientes



Q2

Falso, desde todas las perspectivas.

Bajo Maxwell, el efecto sobre B de actual y mi t son términos separados. Ambos aspectos deben ser considerados.